Added topic tags to some S&J-4 problems + minor typos
[course.git] / latex / problems / Serway_and_Jewett_4 / problem23.01.tex
1 \begin{problem*}{23.1} % induction
2 A flat loop of wire consisting of a single turn of cross-sectional
3 area $A=8.00\U{cm$^2$}$ is perpendicular to a magnetic field that
4 increases uniformly in magnitude from $B_i = 0.500\U{T}$ to $B_f =
5 2.50\U{T}$ in $1.00\U{s}$.  What is the resulting induced current if
6 the loop has a resistance of $R = 2.00\Omega$.
7 \end{problem*}
8
9 \begin{solution}
10 By Faraday's law
11 \begin{equation}
12  \varepsilon = - \frac{d\Phi_B}{dt}
13              = - \frac{(2.0\U{T})\cdot(8.00\E{-4}\U{m$^2$})}{1.00\U{s}}
14              = - 1.6\U{mV} \;.
15 \end{equation}
16
17 By Ohm's law
18 \begin{align}
19  \varepsilon &= V = IR \\
20  I &= \frac{\varepsilon}{R} = \frac{-1.6\U{mV}}{2.00\Omega}
21    = \ans{-0.80\U{mA}} \;.
22 \end{align}
23 \end{solution}
24